Prove the alternating sum of a decreasing sequence converging to 0 is Cauchy. Unicorn Meta Zoo...

Is Bran literally the world's memory?

Where did Arya get these scars?

Reattaching fallen shelf to wall?

Married in secret, can marital status in passport be changed at a later date?

Multiple options vs single option UI

Protagonist's race is hidden - should I reveal it?

Map material from china not allowed to leave the country

Suing a Police Officer Instead of the Police Department

All ASCII characters with a given bit count

Did the Roman Empire have penal colonies?

Raising a bilingual kid. When should we introduce the majority language?

Check if a string is entirely made of the same substring

"Whatever a Russian does, they end up making the Kalashnikov gun"? Are there any similar proverbs in English?

finding a tangent line to a parabola

How to translate "red flag" into Spanish?

Trumpet valves, lengths, and pitch

Do I need to protect SFP ports and optics from dust/contaminants? If so, how?

c++ diamond problem - How to call base method only once

What *exactly* is electrical current, voltage, and resistance?

Retract an already submitted recommendation letter (written for an undergrad student)

Has a Nobel Peace laureate ever been accused of war crimes?

Why did C use the -> operator instead of reusing the . operator?

How to use @AuraEnabled base class method in Lightning Component?

Can I criticise the more senior developers around me for not writing clean code?



Prove the alternating sum of a decreasing sequence converging to 0 is Cauchy.



Unicorn Meta Zoo #1: Why another podcast?
Announcing the arrival of Valued Associate #679: Cesar ManaraSuppose for all positive integers $n$, $|x_n-y_n|< frac{1}{n}$ Prove that $(x_n)$ is also Cauchy.Proof check for completenessProve that $d_n$ is a Cauchy sequence in $mathbb{R}$Prove ${aX_n +bY_n}$ is a Cauchy Sequence.Prove a sequence is a Cauchy and thus convergentIf $(x_n)$ and $(y_n)$ are Cauchy sequences, then give a direct argument that $ (x_n + y_n)$ is a Cauchy sequenceIf ${x_n}$ and ${y_n}$ are Cauchy then $left{frac{2x_n}{y_n}right}$ is CauchyLet ${x_n}$ be a Cauchy sequence of rational numbers. Define a new sequence ${y_n}$ by $y_n = (x_n)(x_{n+1})$. Show that ${y_n}$ is a CS.Let ${x_n}$ be a Cauchy sequence of real numbers, prove that a new sequence ${y_n}$, with $y_n$=$x_n^frac{1}{3}$, is also a Cauchy sequence.$x_n rightarrow x$ iff the modified sequence is Cauchy












2












$begingroup$


Let $(x_n)$ be a decreasing sequence with $x_n > 0$ for all $n in mathbb{N}$, and $(x_n) to 0$. Let $(y_n)$ be defined for all $n in mathbb{N}$ by
$$y_n = x_0 - x_1 + x_2 - cdots + (-1)^n x_n .$$



I want to show, using the $varepsilon$ definition, that $(y_n)$ is Cauchy.



I am trying to find, given $varepsilon > 0$, a real number $N$ such that for all $m$ and $n$ with $m > n > N$, $|y_m - y_n| < varepsilon$.



I have been going backwards to try and find $N$, and have
begin{align*}
|y_m - y_n| & = left| (x_0 - x_1 + cdots pm x_m) - (x_0 - x_1 + cdots pm x_n) right| \
|y_m - y_n| & = left| x_{n + 1} - x_{n + 2} + cdots pm x_{m} right| \
|y_m - y_n| & leq | x_{n + 1} | + | x_{n + 2} | + cdots + | x_{m} | \
|y_m - y_n| & leq ?
end{align*}



I do not know how to get a solution from there, and am not sure about the process, particurlary the last step since I feel getting rid of the minuses might prevent me from finding a solution.










share|cite|improve this question









$endgroup$








  • 1




    $begingroup$
    Because the series is alternating and decreasing, I think you can prove by induction on $m$ that $|y_m-y_n| leq |y_n|$.
    $endgroup$
    – Robert Shore
    3 hours ago










  • $begingroup$
    @RobertShore is my answer okay?
    $endgroup$
    – Subhasis Biswas
    3 hours ago










  • $begingroup$
    @RobertShore yes I can definitely show that, but it brings me to the same issue with $|y_m| leq |x_0 - x_1 + cdots pm x_m|$, and I am unsure how to proceed from there.
    $endgroup$
    – oranji
    38 mins ago


















2












$begingroup$


Let $(x_n)$ be a decreasing sequence with $x_n > 0$ for all $n in mathbb{N}$, and $(x_n) to 0$. Let $(y_n)$ be defined for all $n in mathbb{N}$ by
$$y_n = x_0 - x_1 + x_2 - cdots + (-1)^n x_n .$$



I want to show, using the $varepsilon$ definition, that $(y_n)$ is Cauchy.



I am trying to find, given $varepsilon > 0$, a real number $N$ such that for all $m$ and $n$ with $m > n > N$, $|y_m - y_n| < varepsilon$.



I have been going backwards to try and find $N$, and have
begin{align*}
|y_m - y_n| & = left| (x_0 - x_1 + cdots pm x_m) - (x_0 - x_1 + cdots pm x_n) right| \
|y_m - y_n| & = left| x_{n + 1} - x_{n + 2} + cdots pm x_{m} right| \
|y_m - y_n| & leq | x_{n + 1} | + | x_{n + 2} | + cdots + | x_{m} | \
|y_m - y_n| & leq ?
end{align*}



I do not know how to get a solution from there, and am not sure about the process, particurlary the last step since I feel getting rid of the minuses might prevent me from finding a solution.










share|cite|improve this question









$endgroup$








  • 1




    $begingroup$
    Because the series is alternating and decreasing, I think you can prove by induction on $m$ that $|y_m-y_n| leq |y_n|$.
    $endgroup$
    – Robert Shore
    3 hours ago










  • $begingroup$
    @RobertShore is my answer okay?
    $endgroup$
    – Subhasis Biswas
    3 hours ago










  • $begingroup$
    @RobertShore yes I can definitely show that, but it brings me to the same issue with $|y_m| leq |x_0 - x_1 + cdots pm x_m|$, and I am unsure how to proceed from there.
    $endgroup$
    – oranji
    38 mins ago
















2












2








2


1



$begingroup$


Let $(x_n)$ be a decreasing sequence with $x_n > 0$ for all $n in mathbb{N}$, and $(x_n) to 0$. Let $(y_n)$ be defined for all $n in mathbb{N}$ by
$$y_n = x_0 - x_1 + x_2 - cdots + (-1)^n x_n .$$



I want to show, using the $varepsilon$ definition, that $(y_n)$ is Cauchy.



I am trying to find, given $varepsilon > 0$, a real number $N$ such that for all $m$ and $n$ with $m > n > N$, $|y_m - y_n| < varepsilon$.



I have been going backwards to try and find $N$, and have
begin{align*}
|y_m - y_n| & = left| (x_0 - x_1 + cdots pm x_m) - (x_0 - x_1 + cdots pm x_n) right| \
|y_m - y_n| & = left| x_{n + 1} - x_{n + 2} + cdots pm x_{m} right| \
|y_m - y_n| & leq | x_{n + 1} | + | x_{n + 2} | + cdots + | x_{m} | \
|y_m - y_n| & leq ?
end{align*}



I do not know how to get a solution from there, and am not sure about the process, particurlary the last step since I feel getting rid of the minuses might prevent me from finding a solution.










share|cite|improve this question









$endgroup$




Let $(x_n)$ be a decreasing sequence with $x_n > 0$ for all $n in mathbb{N}$, and $(x_n) to 0$. Let $(y_n)$ be defined for all $n in mathbb{N}$ by
$$y_n = x_0 - x_1 + x_2 - cdots + (-1)^n x_n .$$



I want to show, using the $varepsilon$ definition, that $(y_n)$ is Cauchy.



I am trying to find, given $varepsilon > 0$, a real number $N$ such that for all $m$ and $n$ with $m > n > N$, $|y_m - y_n| < varepsilon$.



I have been going backwards to try and find $N$, and have
begin{align*}
|y_m - y_n| & = left| (x_0 - x_1 + cdots pm x_m) - (x_0 - x_1 + cdots pm x_n) right| \
|y_m - y_n| & = left| x_{n + 1} - x_{n + 2} + cdots pm x_{m} right| \
|y_m - y_n| & leq | x_{n + 1} | + | x_{n + 2} | + cdots + | x_{m} | \
|y_m - y_n| & leq ?
end{align*}



I do not know how to get a solution from there, and am not sure about the process, particurlary the last step since I feel getting rid of the minuses might prevent me from finding a solution.







real-analysis cauchy-sequences






share|cite|improve this question













share|cite|improve this question











share|cite|improve this question




share|cite|improve this question










asked 3 hours ago









oranjioranji

616




616








  • 1




    $begingroup$
    Because the series is alternating and decreasing, I think you can prove by induction on $m$ that $|y_m-y_n| leq |y_n|$.
    $endgroup$
    – Robert Shore
    3 hours ago










  • $begingroup$
    @RobertShore is my answer okay?
    $endgroup$
    – Subhasis Biswas
    3 hours ago










  • $begingroup$
    @RobertShore yes I can definitely show that, but it brings me to the same issue with $|y_m| leq |x_0 - x_1 + cdots pm x_m|$, and I am unsure how to proceed from there.
    $endgroup$
    – oranji
    38 mins ago
















  • 1




    $begingroup$
    Because the series is alternating and decreasing, I think you can prove by induction on $m$ that $|y_m-y_n| leq |y_n|$.
    $endgroup$
    – Robert Shore
    3 hours ago










  • $begingroup$
    @RobertShore is my answer okay?
    $endgroup$
    – Subhasis Biswas
    3 hours ago










  • $begingroup$
    @RobertShore yes I can definitely show that, but it brings me to the same issue with $|y_m| leq |x_0 - x_1 + cdots pm x_m|$, and I am unsure how to proceed from there.
    $endgroup$
    – oranji
    38 mins ago










1




1




$begingroup$
Because the series is alternating and decreasing, I think you can prove by induction on $m$ that $|y_m-y_n| leq |y_n|$.
$endgroup$
– Robert Shore
3 hours ago




$begingroup$
Because the series is alternating and decreasing, I think you can prove by induction on $m$ that $|y_m-y_n| leq |y_n|$.
$endgroup$
– Robert Shore
3 hours ago












$begingroup$
@RobertShore is my answer okay?
$endgroup$
– Subhasis Biswas
3 hours ago




$begingroup$
@RobertShore is my answer okay?
$endgroup$
– Subhasis Biswas
3 hours ago












$begingroup$
@RobertShore yes I can definitely show that, but it brings me to the same issue with $|y_m| leq |x_0 - x_1 + cdots pm x_m|$, and I am unsure how to proceed from there.
$endgroup$
– oranji
38 mins ago






$begingroup$
@RobertShore yes I can definitely show that, but it brings me to the same issue with $|y_m| leq |x_0 - x_1 + cdots pm x_m|$, and I am unsure how to proceed from there.
$endgroup$
– oranji
38 mins ago












2 Answers
2






active

oldest

votes


















2












$begingroup$

To see that the sequence of partial sums is Cauchy, you cannot use the triangle inequality directly as you did. A famous counter example here is $sum_{k=1}^{infty}frac{(-1)^k}{k}$.



What you can do is grouping the terms of the partial sums $s_n= sum_{j=1}^n(-1)^jx_j$ as follows:




  • Let $m = n+k, k,n in mathbb{N}$


Now, you can write $|s_{m} - s_n|$ in two different ways:



$$|s_{n+k} - s_n| = begin{cases}
|x_{n+1} - (x_{n+2}-x_{n+3}) - cdots - (x_{n+2i}-x_{n+2i+1})| & k = 2i+1 \
|x_{n+1} - (x_{n+2}-x_{n+3}) - cdots - (x_{n+2i-2}-x_{n+2i-1}) - x_{2i}| & k = 2i \
end{cases}
$$



$$|s_{n+k} - s_n| = begin{cases}
|(x_{n+1} - x_{n+2}) + cdots + (x_{n+2i-1}-x_{n+2i}) + x_{n+2i+1}| & k = 2i+1 \
|(x_{n+1} - x_{n+2}) + cdots + (x_{n+2i-1}-x_{n+2i}) | & k = 2i \
end{cases}
$$



Using the fact that $x_n searrow 0$, it follows immediately that for all $k in mathbb{N}$ holds
$$|s_{n+k} - s_n| leq x_{n+1}$$



Hence, for $epsilon > 0$ choose $N_{epsilon}$ such that $x_{N_{epsilon}} < epsilon$. Then, for all $m> n > N_{epsilon}$ you have
$$|s_{m} - s_n| leq x_{n+1} leq x_{N_{epsilon}} < epsilon$$






share|cite|improve this answer











$endgroup$













  • $begingroup$
    This is exactly what I was about to do.
    $endgroup$
    – Subhasis Biswas
    37 mins ago






  • 1




    $begingroup$
    @SubhasisBiswas So, I did it for you :-D
    $endgroup$
    – trancelocation
    36 mins ago



















2












$begingroup$

This is also known as the "Leibnitz's Test".



We write $s_n = x_1-x_2+x_3-...+(-1)^{n+1}x_n$



$s_{2n+2}-s_{2n}=u_{2n+1}-u_{2n+2} geq0$ for all $n$.



$s_{2n+1}-s_{2n-1}=-u_{2n}+u_{2n+1} leq 0$



$s_{2n} =u_1 -(u_2-u_3)-(u_4-u_5)...-u_{2n} leq u_1$, i.e. a monotone increasing sequence bounded above.



$s_{2n+1} =(u_1 -u_2)+(u_3-u_4)+...+u_{2n+1} geq u_1-u_2$, i.e. a monotone decreasing sequence bounded below.



Hence, both are convergent subsequences of $(s_n)$. But, we have $lim (s_{2n+1}-s_{2n})=u_{2n+1}=0$, therefore, they converge to the same limit.



Hence, $(s_n)$ converges, i.e. it is Cauchy.



Note: We conclude that $(s_n)$ converges because the indices of the two subsequences $(s_{2n})$ and $(s_{2n+1})$ i.e. $U ={ 2n+1 : n in mathbb{N}}$ and $V ={ 2n : n in mathbb{N}}$ form a partition of $mathbb{N}$ and they both converge to the same limit.






share|cite|improve this answer









$endgroup$













  • $begingroup$
    I want to use the $varepsilon$ definition of a Cauchy sequence, and not the fact that all convergent sequences are Cauchy, which is why I cannot use this solution.
    $endgroup$
    – oranji
    41 mins ago










  • $begingroup$
    I'll edit this answer.
    $endgroup$
    – Subhasis Biswas
    38 mins ago












Your Answer








StackExchange.ready(function() {
var channelOptions = {
tags: "".split(" "),
id: "69"
};
initTagRenderer("".split(" "), "".split(" "), channelOptions);

StackExchange.using("externalEditor", function() {
// Have to fire editor after snippets, if snippets enabled
if (StackExchange.settings.snippets.snippetsEnabled) {
StackExchange.using("snippets", function() {
createEditor();
});
}
else {
createEditor();
}
});

function createEditor() {
StackExchange.prepareEditor({
heartbeatType: 'answer',
autoActivateHeartbeat: false,
convertImagesToLinks: true,
noModals: true,
showLowRepImageUploadWarning: true,
reputationToPostImages: 10,
bindNavPrevention: true,
postfix: "",
imageUploader: {
brandingHtml: "Powered by u003ca class="icon-imgur-white" href="https://imgur.com/"u003eu003c/au003e",
contentPolicyHtml: "User contributions licensed under u003ca href="https://creativecommons.org/licenses/by-sa/3.0/"u003ecc by-sa 3.0 with attribution requiredu003c/au003e u003ca href="https://stackoverflow.com/legal/content-policy"u003e(content policy)u003c/au003e",
allowUrls: true
},
noCode: true, onDemand: true,
discardSelector: ".discard-answer"
,immediatelyShowMarkdownHelp:true
});


}
});














draft saved

draft discarded


















StackExchange.ready(
function () {
StackExchange.openid.initPostLogin('.new-post-login', 'https%3a%2f%2fmath.stackexchange.com%2fquestions%2f3201256%2fprove-the-alternating-sum-of-a-decreasing-sequence-converging-to-0-is-cauchy%23new-answer', 'question_page');
}
);

Post as a guest















Required, but never shown

























2 Answers
2






active

oldest

votes








2 Answers
2






active

oldest

votes









active

oldest

votes






active

oldest

votes









2












$begingroup$

To see that the sequence of partial sums is Cauchy, you cannot use the triangle inequality directly as you did. A famous counter example here is $sum_{k=1}^{infty}frac{(-1)^k}{k}$.



What you can do is grouping the terms of the partial sums $s_n= sum_{j=1}^n(-1)^jx_j$ as follows:




  • Let $m = n+k, k,n in mathbb{N}$


Now, you can write $|s_{m} - s_n|$ in two different ways:



$$|s_{n+k} - s_n| = begin{cases}
|x_{n+1} - (x_{n+2}-x_{n+3}) - cdots - (x_{n+2i}-x_{n+2i+1})| & k = 2i+1 \
|x_{n+1} - (x_{n+2}-x_{n+3}) - cdots - (x_{n+2i-2}-x_{n+2i-1}) - x_{2i}| & k = 2i \
end{cases}
$$



$$|s_{n+k} - s_n| = begin{cases}
|(x_{n+1} - x_{n+2}) + cdots + (x_{n+2i-1}-x_{n+2i}) + x_{n+2i+1}| & k = 2i+1 \
|(x_{n+1} - x_{n+2}) + cdots + (x_{n+2i-1}-x_{n+2i}) | & k = 2i \
end{cases}
$$



Using the fact that $x_n searrow 0$, it follows immediately that for all $k in mathbb{N}$ holds
$$|s_{n+k} - s_n| leq x_{n+1}$$



Hence, for $epsilon > 0$ choose $N_{epsilon}$ such that $x_{N_{epsilon}} < epsilon$. Then, for all $m> n > N_{epsilon}$ you have
$$|s_{m} - s_n| leq x_{n+1} leq x_{N_{epsilon}} < epsilon$$






share|cite|improve this answer











$endgroup$













  • $begingroup$
    This is exactly what I was about to do.
    $endgroup$
    – Subhasis Biswas
    37 mins ago






  • 1




    $begingroup$
    @SubhasisBiswas So, I did it for you :-D
    $endgroup$
    – trancelocation
    36 mins ago
















2












$begingroup$

To see that the sequence of partial sums is Cauchy, you cannot use the triangle inequality directly as you did. A famous counter example here is $sum_{k=1}^{infty}frac{(-1)^k}{k}$.



What you can do is grouping the terms of the partial sums $s_n= sum_{j=1}^n(-1)^jx_j$ as follows:




  • Let $m = n+k, k,n in mathbb{N}$


Now, you can write $|s_{m} - s_n|$ in two different ways:



$$|s_{n+k} - s_n| = begin{cases}
|x_{n+1} - (x_{n+2}-x_{n+3}) - cdots - (x_{n+2i}-x_{n+2i+1})| & k = 2i+1 \
|x_{n+1} - (x_{n+2}-x_{n+3}) - cdots - (x_{n+2i-2}-x_{n+2i-1}) - x_{2i}| & k = 2i \
end{cases}
$$



$$|s_{n+k} - s_n| = begin{cases}
|(x_{n+1} - x_{n+2}) + cdots + (x_{n+2i-1}-x_{n+2i}) + x_{n+2i+1}| & k = 2i+1 \
|(x_{n+1} - x_{n+2}) + cdots + (x_{n+2i-1}-x_{n+2i}) | & k = 2i \
end{cases}
$$



Using the fact that $x_n searrow 0$, it follows immediately that for all $k in mathbb{N}$ holds
$$|s_{n+k} - s_n| leq x_{n+1}$$



Hence, for $epsilon > 0$ choose $N_{epsilon}$ such that $x_{N_{epsilon}} < epsilon$. Then, for all $m> n > N_{epsilon}$ you have
$$|s_{m} - s_n| leq x_{n+1} leq x_{N_{epsilon}} < epsilon$$






share|cite|improve this answer











$endgroup$













  • $begingroup$
    This is exactly what I was about to do.
    $endgroup$
    – Subhasis Biswas
    37 mins ago






  • 1




    $begingroup$
    @SubhasisBiswas So, I did it for you :-D
    $endgroup$
    – trancelocation
    36 mins ago














2












2








2





$begingroup$

To see that the sequence of partial sums is Cauchy, you cannot use the triangle inequality directly as you did. A famous counter example here is $sum_{k=1}^{infty}frac{(-1)^k}{k}$.



What you can do is grouping the terms of the partial sums $s_n= sum_{j=1}^n(-1)^jx_j$ as follows:




  • Let $m = n+k, k,n in mathbb{N}$


Now, you can write $|s_{m} - s_n|$ in two different ways:



$$|s_{n+k} - s_n| = begin{cases}
|x_{n+1} - (x_{n+2}-x_{n+3}) - cdots - (x_{n+2i}-x_{n+2i+1})| & k = 2i+1 \
|x_{n+1} - (x_{n+2}-x_{n+3}) - cdots - (x_{n+2i-2}-x_{n+2i-1}) - x_{2i}| & k = 2i \
end{cases}
$$



$$|s_{n+k} - s_n| = begin{cases}
|(x_{n+1} - x_{n+2}) + cdots + (x_{n+2i-1}-x_{n+2i}) + x_{n+2i+1}| & k = 2i+1 \
|(x_{n+1} - x_{n+2}) + cdots + (x_{n+2i-1}-x_{n+2i}) | & k = 2i \
end{cases}
$$



Using the fact that $x_n searrow 0$, it follows immediately that for all $k in mathbb{N}$ holds
$$|s_{n+k} - s_n| leq x_{n+1}$$



Hence, for $epsilon > 0$ choose $N_{epsilon}$ such that $x_{N_{epsilon}} < epsilon$. Then, for all $m> n > N_{epsilon}$ you have
$$|s_{m} - s_n| leq x_{n+1} leq x_{N_{epsilon}} < epsilon$$






share|cite|improve this answer











$endgroup$



To see that the sequence of partial sums is Cauchy, you cannot use the triangle inequality directly as you did. A famous counter example here is $sum_{k=1}^{infty}frac{(-1)^k}{k}$.



What you can do is grouping the terms of the partial sums $s_n= sum_{j=1}^n(-1)^jx_j$ as follows:




  • Let $m = n+k, k,n in mathbb{N}$


Now, you can write $|s_{m} - s_n|$ in two different ways:



$$|s_{n+k} - s_n| = begin{cases}
|x_{n+1} - (x_{n+2}-x_{n+3}) - cdots - (x_{n+2i}-x_{n+2i+1})| & k = 2i+1 \
|x_{n+1} - (x_{n+2}-x_{n+3}) - cdots - (x_{n+2i-2}-x_{n+2i-1}) - x_{2i}| & k = 2i \
end{cases}
$$



$$|s_{n+k} - s_n| = begin{cases}
|(x_{n+1} - x_{n+2}) + cdots + (x_{n+2i-1}-x_{n+2i}) + x_{n+2i+1}| & k = 2i+1 \
|(x_{n+1} - x_{n+2}) + cdots + (x_{n+2i-1}-x_{n+2i}) | & k = 2i \
end{cases}
$$



Using the fact that $x_n searrow 0$, it follows immediately that for all $k in mathbb{N}$ holds
$$|s_{n+k} - s_n| leq x_{n+1}$$



Hence, for $epsilon > 0$ choose $N_{epsilon}$ such that $x_{N_{epsilon}} < epsilon$. Then, for all $m> n > N_{epsilon}$ you have
$$|s_{m} - s_n| leq x_{n+1} leq x_{N_{epsilon}} < epsilon$$







share|cite|improve this answer














share|cite|improve this answer



share|cite|improve this answer








edited 21 mins ago

























answered 38 mins ago









trancelocationtrancelocation

14.6k1929




14.6k1929












  • $begingroup$
    This is exactly what I was about to do.
    $endgroup$
    – Subhasis Biswas
    37 mins ago






  • 1




    $begingroup$
    @SubhasisBiswas So, I did it for you :-D
    $endgroup$
    – trancelocation
    36 mins ago


















  • $begingroup$
    This is exactly what I was about to do.
    $endgroup$
    – Subhasis Biswas
    37 mins ago






  • 1




    $begingroup$
    @SubhasisBiswas So, I did it for you :-D
    $endgroup$
    – trancelocation
    36 mins ago
















$begingroup$
This is exactly what I was about to do.
$endgroup$
– Subhasis Biswas
37 mins ago




$begingroup$
This is exactly what I was about to do.
$endgroup$
– Subhasis Biswas
37 mins ago




1




1




$begingroup$
@SubhasisBiswas So, I did it for you :-D
$endgroup$
– trancelocation
36 mins ago




$begingroup$
@SubhasisBiswas So, I did it for you :-D
$endgroup$
– trancelocation
36 mins ago











2












$begingroup$

This is also known as the "Leibnitz's Test".



We write $s_n = x_1-x_2+x_3-...+(-1)^{n+1}x_n$



$s_{2n+2}-s_{2n}=u_{2n+1}-u_{2n+2} geq0$ for all $n$.



$s_{2n+1}-s_{2n-1}=-u_{2n}+u_{2n+1} leq 0$



$s_{2n} =u_1 -(u_2-u_3)-(u_4-u_5)...-u_{2n} leq u_1$, i.e. a monotone increasing sequence bounded above.



$s_{2n+1} =(u_1 -u_2)+(u_3-u_4)+...+u_{2n+1} geq u_1-u_2$, i.e. a monotone decreasing sequence bounded below.



Hence, both are convergent subsequences of $(s_n)$. But, we have $lim (s_{2n+1}-s_{2n})=u_{2n+1}=0$, therefore, they converge to the same limit.



Hence, $(s_n)$ converges, i.e. it is Cauchy.



Note: We conclude that $(s_n)$ converges because the indices of the two subsequences $(s_{2n})$ and $(s_{2n+1})$ i.e. $U ={ 2n+1 : n in mathbb{N}}$ and $V ={ 2n : n in mathbb{N}}$ form a partition of $mathbb{N}$ and they both converge to the same limit.






share|cite|improve this answer









$endgroup$













  • $begingroup$
    I want to use the $varepsilon$ definition of a Cauchy sequence, and not the fact that all convergent sequences are Cauchy, which is why I cannot use this solution.
    $endgroup$
    – oranji
    41 mins ago










  • $begingroup$
    I'll edit this answer.
    $endgroup$
    – Subhasis Biswas
    38 mins ago
















2












$begingroup$

This is also known as the "Leibnitz's Test".



We write $s_n = x_1-x_2+x_3-...+(-1)^{n+1}x_n$



$s_{2n+2}-s_{2n}=u_{2n+1}-u_{2n+2} geq0$ for all $n$.



$s_{2n+1}-s_{2n-1}=-u_{2n}+u_{2n+1} leq 0$



$s_{2n} =u_1 -(u_2-u_3)-(u_4-u_5)...-u_{2n} leq u_1$, i.e. a monotone increasing sequence bounded above.



$s_{2n+1} =(u_1 -u_2)+(u_3-u_4)+...+u_{2n+1} geq u_1-u_2$, i.e. a monotone decreasing sequence bounded below.



Hence, both are convergent subsequences of $(s_n)$. But, we have $lim (s_{2n+1}-s_{2n})=u_{2n+1}=0$, therefore, they converge to the same limit.



Hence, $(s_n)$ converges, i.e. it is Cauchy.



Note: We conclude that $(s_n)$ converges because the indices of the two subsequences $(s_{2n})$ and $(s_{2n+1})$ i.e. $U ={ 2n+1 : n in mathbb{N}}$ and $V ={ 2n : n in mathbb{N}}$ form a partition of $mathbb{N}$ and they both converge to the same limit.






share|cite|improve this answer









$endgroup$













  • $begingroup$
    I want to use the $varepsilon$ definition of a Cauchy sequence, and not the fact that all convergent sequences are Cauchy, which is why I cannot use this solution.
    $endgroup$
    – oranji
    41 mins ago










  • $begingroup$
    I'll edit this answer.
    $endgroup$
    – Subhasis Biswas
    38 mins ago














2












2








2





$begingroup$

This is also known as the "Leibnitz's Test".



We write $s_n = x_1-x_2+x_3-...+(-1)^{n+1}x_n$



$s_{2n+2}-s_{2n}=u_{2n+1}-u_{2n+2} geq0$ for all $n$.



$s_{2n+1}-s_{2n-1}=-u_{2n}+u_{2n+1} leq 0$



$s_{2n} =u_1 -(u_2-u_3)-(u_4-u_5)...-u_{2n} leq u_1$, i.e. a monotone increasing sequence bounded above.



$s_{2n+1} =(u_1 -u_2)+(u_3-u_4)+...+u_{2n+1} geq u_1-u_2$, i.e. a monotone decreasing sequence bounded below.



Hence, both are convergent subsequences of $(s_n)$. But, we have $lim (s_{2n+1}-s_{2n})=u_{2n+1}=0$, therefore, they converge to the same limit.



Hence, $(s_n)$ converges, i.e. it is Cauchy.



Note: We conclude that $(s_n)$ converges because the indices of the two subsequences $(s_{2n})$ and $(s_{2n+1})$ i.e. $U ={ 2n+1 : n in mathbb{N}}$ and $V ={ 2n : n in mathbb{N}}$ form a partition of $mathbb{N}$ and they both converge to the same limit.






share|cite|improve this answer









$endgroup$



This is also known as the "Leibnitz's Test".



We write $s_n = x_1-x_2+x_3-...+(-1)^{n+1}x_n$



$s_{2n+2}-s_{2n}=u_{2n+1}-u_{2n+2} geq0$ for all $n$.



$s_{2n+1}-s_{2n-1}=-u_{2n}+u_{2n+1} leq 0$



$s_{2n} =u_1 -(u_2-u_3)-(u_4-u_5)...-u_{2n} leq u_1$, i.e. a monotone increasing sequence bounded above.



$s_{2n+1} =(u_1 -u_2)+(u_3-u_4)+...+u_{2n+1} geq u_1-u_2$, i.e. a monotone decreasing sequence bounded below.



Hence, both are convergent subsequences of $(s_n)$. But, we have $lim (s_{2n+1}-s_{2n})=u_{2n+1}=0$, therefore, they converge to the same limit.



Hence, $(s_n)$ converges, i.e. it is Cauchy.



Note: We conclude that $(s_n)$ converges because the indices of the two subsequences $(s_{2n})$ and $(s_{2n+1})$ i.e. $U ={ 2n+1 : n in mathbb{N}}$ and $V ={ 2n : n in mathbb{N}}$ form a partition of $mathbb{N}$ and they both converge to the same limit.







share|cite|improve this answer












share|cite|improve this answer



share|cite|improve this answer










answered 3 hours ago









Subhasis BiswasSubhasis Biswas

608512




608512












  • $begingroup$
    I want to use the $varepsilon$ definition of a Cauchy sequence, and not the fact that all convergent sequences are Cauchy, which is why I cannot use this solution.
    $endgroup$
    – oranji
    41 mins ago










  • $begingroup$
    I'll edit this answer.
    $endgroup$
    – Subhasis Biswas
    38 mins ago


















  • $begingroup$
    I want to use the $varepsilon$ definition of a Cauchy sequence, and not the fact that all convergent sequences are Cauchy, which is why I cannot use this solution.
    $endgroup$
    – oranji
    41 mins ago










  • $begingroup$
    I'll edit this answer.
    $endgroup$
    – Subhasis Biswas
    38 mins ago
















$begingroup$
I want to use the $varepsilon$ definition of a Cauchy sequence, and not the fact that all convergent sequences are Cauchy, which is why I cannot use this solution.
$endgroup$
– oranji
41 mins ago




$begingroup$
I want to use the $varepsilon$ definition of a Cauchy sequence, and not the fact that all convergent sequences are Cauchy, which is why I cannot use this solution.
$endgroup$
– oranji
41 mins ago












$begingroup$
I'll edit this answer.
$endgroup$
– Subhasis Biswas
38 mins ago




$begingroup$
I'll edit this answer.
$endgroup$
– Subhasis Biswas
38 mins ago


















draft saved

draft discarded




















































Thanks for contributing an answer to Mathematics Stack Exchange!


  • Please be sure to answer the question. Provide details and share your research!

But avoid



  • Asking for help, clarification, or responding to other answers.

  • Making statements based on opinion; back them up with references or personal experience.


Use MathJax to format equations. MathJax reference.


To learn more, see our tips on writing great answers.




draft saved


draft discarded














StackExchange.ready(
function () {
StackExchange.openid.initPostLogin('.new-post-login', 'https%3a%2f%2fmath.stackexchange.com%2fquestions%2f3201256%2fprove-the-alternating-sum-of-a-decreasing-sequence-converging-to-0-is-cauchy%23new-answer', 'question_page');
}
);

Post as a guest















Required, but never shown





















































Required, but never shown














Required, but never shown












Required, but never shown







Required, but never shown

































Required, but never shown














Required, but never shown












Required, but never shown







Required, but never shown







Popular posts from this blog

Gersau Kjelder | Navigasjonsmeny46°59′0″N 8°31′0″E46°59′0″N...

Hestehale Innhaldsliste Hestehale på kvinner | Hestehale på menn | Galleri | Sjå òg |...

What is the “three and three hundred thousand syndrome”?Who wrote the book Arena?What five creatures were...